Difference between revisions of "Forest UCM LEq"

From New IAC Wiki
Jump to navigation Jump to search
Line 28: Line 28:
  
 
<math>  \frac{d}{dt} \left ( \frac{\partial \mathcal {L} }{\partial x^{\prime}} \right ) =  \frac{d}{dt} \left ( \frac{\partial }{\partial x^{\prime}} \right ) \left ( T(\dot x) - U(x) \right ) </math>
 
<math>  \frac{d}{dt} \left ( \frac{\partial \mathcal {L} }{\partial x^{\prime}} \right ) =  \frac{d}{dt} \left ( \frac{\partial }{\partial x^{\prime}} \right ) \left ( T(\dot x) - U(x) \right ) </math>
 +
<math>  =  \frac{d}{dt} \left ( \frac{\partial }{\partial x^{\prime}} \right )  T(\dot x) </math>
  
  

Revision as of 19:41, 24 October 2014

Lagrange's Equations

Lagrange's equations

Hamilton's principle falls out of the calculus of variations in that seeking the shortest time interval is the focus of the variations.


Of all possible paths along which a dynamical system may move from on point to another within a specified time interval, the actual path followed is that which minimizes the time integral of the difference between the kinetic and potential energies.

Casting this in the language of the calculus of variations

[math]S \equiv \int_{t_1}^{t_2} \left ( T(\dot x) - U(x) \right ) dt = \int_{t_1}^{t_2} f(x,\dot x ; t) dt = \int_{t_1}^{t_2} \mathcal {L} ( x, \dot x;t) dt [/math]

if you want the above "action" integral to be stationary then according to the calculus of variations you want the Euler-Lagrange equation to be satisfied where

[math] \left [ \left ( \frac{\partial f}{\partial x} \right ) - \frac{d}{dt} \left ( \frac{\partial f}{\partial x^{\prime}} \right ) \right ] =0 [/math]


or

[math] \left [ \left ( \frac{\partial \mathcal {L} }{\partial x} \right ) - \frac{d}{dt} \left ( \frac{\partial \mathcal {L} }{\partial x^{\prime}} \right ) \right ] =0 [/math]


[math] \left ( \frac{\partial \mathcal {L} }{\partial x} \right ) = \frac{\partial }{\partial x} \left ( T(\dot x) - U(x) \right ) = -\frac{\partial U(x)}{\partial x} = F_x[/math]


[math] \frac{d}{dt} \left ( \frac{\partial \mathcal {L} }{\partial x^{\prime}} \right ) = \frac{d}{dt} \left ( \frac{\partial }{\partial x^{\prime}} \right ) \left ( T(\dot x) - U(x) \right ) [/math] [math] = \frac{d}{dt} \left ( \frac{\partial }{\partial x^{\prime}} \right ) T(\dot x) [/math]


We saw the application of this principle in the previous chapter's example of the brachyostochrome problem where a path was determine for a bead falling from point A to point B in the least amount of time.


Forest_Ugrad_ClassicalMechanics